LSAT and Law School Admissions Forum

Get expert LSAT preparation and law school admissions advice from PowerScore Test Preparation.

User avatar
 Dave Killoran
PowerScore Staff
  • PowerScore Staff
  • Posts: 5853
  • Joined: Mar 25, 2011
|
#84898
Complete Question Explanation
(The complete setup for this game can be found here: lsat/viewtopic.php?t=8333)

The correct answer choice is (C)

If F is third, then from the combination of the first two rules, J, T, and H must be more recent, and occupy the fourth, fifth, and sixth positions, in some order:
PT61_O2010 LG Explanations-G2Q9.png
The third rule now comes into play, and Possibility #1 must be in effect. In that possibility,
PT61_O2010 LG Explanations-G2Q9D2.png
and as the first two positions are all that remain for P and N, N must be second. Consequently, answer choice (C) is correct.
You do not have the required permissions to view the files attached to this post.

Get the most out of your LSAT Prep Plus subscription.

Analyze and track your performance with our Testing and Analytics Package.